How to Correctly Calculate the Line Integral Over a Semicircular Path?

Click For Summary
To correctly evaluate the line integral over the semicircular path, the parametric equation should be set as r(t) = (2cos(t), 2sin(t)), reflecting the correct radius of 2 for the circle x^2 + y^2 = 4. The limits of integration for the right half of the circle should be from -π/2 to π/2. The integrand is then adjusted to (2)(cos(t))(sin(t))^4. The initial calculations led to an incorrect answer of 8/5, which was due to using the wrong radius and limits. Ensuring the correct parameters is crucial for obtaining the accurate result.
fk378
Messages
366
Reaction score
0

Homework Statement


Evaluate the line integral, where C is the given curve.
integral of xy^4. C is the right half of the circle x^2 + y^2 =16

Homework Equations


integral of line integral= integral of r(t) |r'(t)| dt

The Attempt at a Solution


I set up a parametric equation to be r(t)=(4cost, 4sint)
then r'(t)=(-4sint, 4cost) --> |r'(t)|=4
My t is between -pi/2 and pi/2---these are my limits of integration.

With the limits of integration, my integrand is (4)(cost)(sint)^4. Setting my u=sint, my final answer came out to be 8/5. I know I got the wrong answer though...
 
Last edited:
Physics news on Phys.org
fk378 said:

Homework Statement


Evaluate the line integral, where C is the given curve.
integral of xy^4. C is the right half of the circle x^2 + y^2 =4

Homework Equations


integral of line integral= integral of r(t) |r'(t)| dt


The Attempt at a Solution


I set up a parametric equation to be r(t)=(4cost, 4sint)
then r'(t)=(-4sint, 4cost) --> |r'(t)|=4
My t is between -pi/4 and pi/4---these are my limits of integration.

With the limits of integration, my integrand is (4)(cost)(sint)^4. Setting my u=sint, my final answer came out to be 8/5. I know I got the wrong answer though...

Why did you use pi/4 and -pi/4?? If you you plug those values in your parametrized curve, you don't get the limits corresponding to the right half of a circle! Or am I missing something?
 
Well isn't the right half of the circle just when x>0? So I thought it would correspond to all values of y...
 
fk378 said:
Well isn't the right half of the circle just when x>0?
Right
So I thought it would correspond to all values of y...

Your parametrized curve is 4 cos(t), 4 sin(t)

If you plug t = pi/4 in that what do you get?
 
Ah, sorry. I mean to say -pi/2 to pi/2. I just fixed it.
 
fk378 said:
Ah, sorry. I mean to say -pi/2 to pi/2. I just fixed it.

Ok. The other error is that the radius is 2, not 4. So you must changed your parametrized curve
 
Question: A clock's minute hand has length 4 and its hour hand has length 3. What is the distance between the tips at the moment when it is increasing most rapidly?(Putnam Exam Question) Answer: Making assumption that both the hands moves at constant angular velocities, the answer is ## \sqrt{7} .## But don't you think this assumption is somewhat doubtful and wrong?

Similar threads

  • · Replies 9 ·
Replies
9
Views
1K
Replies
12
Views
2K
Replies
6
Views
3K
Replies
3
Views
2K
  • · Replies 2 ·
Replies
2
Views
1K
  • · Replies 12 ·
Replies
12
Views
2K
  • · Replies 3 ·
Replies
3
Views
2K
  • · Replies 7 ·
Replies
7
Views
2K
  • · Replies 6 ·
Replies
6
Views
1K
Replies
10
Views
3K